Suppose H⩽G, prove that if (H,G′)=⟨e⟩, then (H′,G)=⟨e⟩.











up vote
2
down vote

favorite












I know this is a duplicate of Suppose $ Hleqslant G $, prove that if $ (H, G')=langle e rangle $, then $ (H', G)=langle e rangle $. but the only answer which does not use the Three Subgroups Lemma (which, at this point, Hungerford has not covered) did not make sense to me, and since my reputation is still low, I can't leave a comment asking for clarification. The given solution is as follows:



"By the previous exercise in the book, for $h,k∈H$, and $g∈G$, we have $[hk,g]=h[k,g]h^{−1}[h,g]=[k,g][h,g],$ because $(H′,G)=1$ (I am writing 1 for ⟨e⟩ and also for e.)



We have to prove that $[[h,k],g]=1$. We have $$[[h,k],g]=[hkh^{−1}k^{−1},g]=[k^{−1},g][h^{−1},g][k,g][h,g]=[k,g]^{−1}[h,g]^{−1}[k,g][h,g].$$



Now, using $(H′,G)=1$ again, we have $h^{−1}[k,g]h=[k,g]$ and $hg^{−1}[k,g]gh^{−1}=g^{−1}[k,g]g,$ and so



$$[k,g]^{−1}[h,g]^{−1}[k,g][h,g]=[k,g]^{−1}ghg^{−1}h^{−1}[k,g]hgh^{−1}g^{−1}=[k,g]^{−1}[k,g]=1$$
as required."



The problem I have with this proof is that they seem to be using $(H',G)=1$, which is what we're trying to prove; we have that $(H,G')=1$ and we're trying to prove $(H',G)=1$, so it seems to me we shouldn't assume that in our proof. Am I missing something? If not, and this proof is invalid, would anyone be able to provide a valid proof of this (without referring to the Three Subgroups Lemma)?










share|cite|improve this question


















  • 1




    Sorry, the proof was mine, and when I wrote "using $(H',G)=1$", I meant "using $(H,G')=1$", which we are assuming. So it was just a typo, both times. I have edited the solution now to remove the mistake.
    – Derek Holt
    Nov 18 at 11:59












  • Gotcha! Thanks for the edit, I am able to follow most of the proof now. I do have one more question, though; How did you get that $[k^{−1},g][h^{−1},g][k,g][h,g]=[k,g]^{−1}[h,g]^{−1}[k,g][h,g]$? I wrote it out and it doesn't seem to be equal at first glance; were you using something to get this equivalence?
    – John
    Nov 18 at 19:30










  • $[k^{-1},g]=[k,g]^{-1}$ follows from $[hk,g]=[k,g][h,g]$, which we have just proved.
    – Derek Holt
    Nov 18 at 22:18

















up vote
2
down vote

favorite












I know this is a duplicate of Suppose $ Hleqslant G $, prove that if $ (H, G')=langle e rangle $, then $ (H', G)=langle e rangle $. but the only answer which does not use the Three Subgroups Lemma (which, at this point, Hungerford has not covered) did not make sense to me, and since my reputation is still low, I can't leave a comment asking for clarification. The given solution is as follows:



"By the previous exercise in the book, for $h,k∈H$, and $g∈G$, we have $[hk,g]=h[k,g]h^{−1}[h,g]=[k,g][h,g],$ because $(H′,G)=1$ (I am writing 1 for ⟨e⟩ and also for e.)



We have to prove that $[[h,k],g]=1$. We have $$[[h,k],g]=[hkh^{−1}k^{−1},g]=[k^{−1},g][h^{−1},g][k,g][h,g]=[k,g]^{−1}[h,g]^{−1}[k,g][h,g].$$



Now, using $(H′,G)=1$ again, we have $h^{−1}[k,g]h=[k,g]$ and $hg^{−1}[k,g]gh^{−1}=g^{−1}[k,g]g,$ and so



$$[k,g]^{−1}[h,g]^{−1}[k,g][h,g]=[k,g]^{−1}ghg^{−1}h^{−1}[k,g]hgh^{−1}g^{−1}=[k,g]^{−1}[k,g]=1$$
as required."



The problem I have with this proof is that they seem to be using $(H',G)=1$, which is what we're trying to prove; we have that $(H,G')=1$ and we're trying to prove $(H',G)=1$, so it seems to me we shouldn't assume that in our proof. Am I missing something? If not, and this proof is invalid, would anyone be able to provide a valid proof of this (without referring to the Three Subgroups Lemma)?










share|cite|improve this question


















  • 1




    Sorry, the proof was mine, and when I wrote "using $(H',G)=1$", I meant "using $(H,G')=1$", which we are assuming. So it was just a typo, both times. I have edited the solution now to remove the mistake.
    – Derek Holt
    Nov 18 at 11:59












  • Gotcha! Thanks for the edit, I am able to follow most of the proof now. I do have one more question, though; How did you get that $[k^{−1},g][h^{−1},g][k,g][h,g]=[k,g]^{−1}[h,g]^{−1}[k,g][h,g]$? I wrote it out and it doesn't seem to be equal at first glance; were you using something to get this equivalence?
    – John
    Nov 18 at 19:30










  • $[k^{-1},g]=[k,g]^{-1}$ follows from $[hk,g]=[k,g][h,g]$, which we have just proved.
    – Derek Holt
    Nov 18 at 22:18















up vote
2
down vote

favorite









up vote
2
down vote

favorite











I know this is a duplicate of Suppose $ Hleqslant G $, prove that if $ (H, G')=langle e rangle $, then $ (H', G)=langle e rangle $. but the only answer which does not use the Three Subgroups Lemma (which, at this point, Hungerford has not covered) did not make sense to me, and since my reputation is still low, I can't leave a comment asking for clarification. The given solution is as follows:



"By the previous exercise in the book, for $h,k∈H$, and $g∈G$, we have $[hk,g]=h[k,g]h^{−1}[h,g]=[k,g][h,g],$ because $(H′,G)=1$ (I am writing 1 for ⟨e⟩ and also for e.)



We have to prove that $[[h,k],g]=1$. We have $$[[h,k],g]=[hkh^{−1}k^{−1},g]=[k^{−1},g][h^{−1},g][k,g][h,g]=[k,g]^{−1}[h,g]^{−1}[k,g][h,g].$$



Now, using $(H′,G)=1$ again, we have $h^{−1}[k,g]h=[k,g]$ and $hg^{−1}[k,g]gh^{−1}=g^{−1}[k,g]g,$ and so



$$[k,g]^{−1}[h,g]^{−1}[k,g][h,g]=[k,g]^{−1}ghg^{−1}h^{−1}[k,g]hgh^{−1}g^{−1}=[k,g]^{−1}[k,g]=1$$
as required."



The problem I have with this proof is that they seem to be using $(H',G)=1$, which is what we're trying to prove; we have that $(H,G')=1$ and we're trying to prove $(H',G)=1$, so it seems to me we shouldn't assume that in our proof. Am I missing something? If not, and this proof is invalid, would anyone be able to provide a valid proof of this (without referring to the Three Subgroups Lemma)?










share|cite|improve this question













I know this is a duplicate of Suppose $ Hleqslant G $, prove that if $ (H, G')=langle e rangle $, then $ (H', G)=langle e rangle $. but the only answer which does not use the Three Subgroups Lemma (which, at this point, Hungerford has not covered) did not make sense to me, and since my reputation is still low, I can't leave a comment asking for clarification. The given solution is as follows:



"By the previous exercise in the book, for $h,k∈H$, and $g∈G$, we have $[hk,g]=h[k,g]h^{−1}[h,g]=[k,g][h,g],$ because $(H′,G)=1$ (I am writing 1 for ⟨e⟩ and also for e.)



We have to prove that $[[h,k],g]=1$. We have $$[[h,k],g]=[hkh^{−1}k^{−1},g]=[k^{−1},g][h^{−1},g][k,g][h,g]=[k,g]^{−1}[h,g]^{−1}[k,g][h,g].$$



Now, using $(H′,G)=1$ again, we have $h^{−1}[k,g]h=[k,g]$ and $hg^{−1}[k,g]gh^{−1}=g^{−1}[k,g]g,$ and so



$$[k,g]^{−1}[h,g]^{−1}[k,g][h,g]=[k,g]^{−1}ghg^{−1}h^{−1}[k,g]hgh^{−1}g^{−1}=[k,g]^{−1}[k,g]=1$$
as required."



The problem I have with this proof is that they seem to be using $(H',G)=1$, which is what we're trying to prove; we have that $(H,G')=1$ and we're trying to prove $(H',G)=1$, so it seems to me we shouldn't assume that in our proof. Am I missing something? If not, and this proof is invalid, would anyone be able to provide a valid proof of this (without referring to the Three Subgroups Lemma)?







abstract-algebra group-theory






share|cite|improve this question













share|cite|improve this question











share|cite|improve this question




share|cite|improve this question










asked Nov 18 at 2:19









John

356




356








  • 1




    Sorry, the proof was mine, and when I wrote "using $(H',G)=1$", I meant "using $(H,G')=1$", which we are assuming. So it was just a typo, both times. I have edited the solution now to remove the mistake.
    – Derek Holt
    Nov 18 at 11:59












  • Gotcha! Thanks for the edit, I am able to follow most of the proof now. I do have one more question, though; How did you get that $[k^{−1},g][h^{−1},g][k,g][h,g]=[k,g]^{−1}[h,g]^{−1}[k,g][h,g]$? I wrote it out and it doesn't seem to be equal at first glance; were you using something to get this equivalence?
    – John
    Nov 18 at 19:30










  • $[k^{-1},g]=[k,g]^{-1}$ follows from $[hk,g]=[k,g][h,g]$, which we have just proved.
    – Derek Holt
    Nov 18 at 22:18
















  • 1




    Sorry, the proof was mine, and when I wrote "using $(H',G)=1$", I meant "using $(H,G')=1$", which we are assuming. So it was just a typo, both times. I have edited the solution now to remove the mistake.
    – Derek Holt
    Nov 18 at 11:59












  • Gotcha! Thanks for the edit, I am able to follow most of the proof now. I do have one more question, though; How did you get that $[k^{−1},g][h^{−1},g][k,g][h,g]=[k,g]^{−1}[h,g]^{−1}[k,g][h,g]$? I wrote it out and it doesn't seem to be equal at first glance; were you using something to get this equivalence?
    – John
    Nov 18 at 19:30










  • $[k^{-1},g]=[k,g]^{-1}$ follows from $[hk,g]=[k,g][h,g]$, which we have just proved.
    – Derek Holt
    Nov 18 at 22:18










1




1




Sorry, the proof was mine, and when I wrote "using $(H',G)=1$", I meant "using $(H,G')=1$", which we are assuming. So it was just a typo, both times. I have edited the solution now to remove the mistake.
– Derek Holt
Nov 18 at 11:59






Sorry, the proof was mine, and when I wrote "using $(H',G)=1$", I meant "using $(H,G')=1$", which we are assuming. So it was just a typo, both times. I have edited the solution now to remove the mistake.
– Derek Holt
Nov 18 at 11:59














Gotcha! Thanks for the edit, I am able to follow most of the proof now. I do have one more question, though; How did you get that $[k^{−1},g][h^{−1},g][k,g][h,g]=[k,g]^{−1}[h,g]^{−1}[k,g][h,g]$? I wrote it out and it doesn't seem to be equal at first glance; were you using something to get this equivalence?
– John
Nov 18 at 19:30




Gotcha! Thanks for the edit, I am able to follow most of the proof now. I do have one more question, though; How did you get that $[k^{−1},g][h^{−1},g][k,g][h,g]=[k,g]^{−1}[h,g]^{−1}[k,g][h,g]$? I wrote it out and it doesn't seem to be equal at first glance; were you using something to get this equivalence?
– John
Nov 18 at 19:30












$[k^{-1},g]=[k,g]^{-1}$ follows from $[hk,g]=[k,g][h,g]$, which we have just proved.
– Derek Holt
Nov 18 at 22:18






$[k^{-1},g]=[k,g]^{-1}$ follows from $[hk,g]=[k,g][h,g]$, which we have just proved.
– Derek Holt
Nov 18 at 22:18

















active

oldest

votes











Your Answer





StackExchange.ifUsing("editor", function () {
return StackExchange.using("mathjaxEditing", function () {
StackExchange.MarkdownEditor.creationCallbacks.add(function (editor, postfix) {
StackExchange.mathjaxEditing.prepareWmdForMathJax(editor, postfix, [["$", "$"], ["\\(","\\)"]]);
});
});
}, "mathjax-editing");

StackExchange.ready(function() {
var channelOptions = {
tags: "".split(" "),
id: "69"
};
initTagRenderer("".split(" "), "".split(" "), channelOptions);

StackExchange.using("externalEditor", function() {
// Have to fire editor after snippets, if snippets enabled
if (StackExchange.settings.snippets.snippetsEnabled) {
StackExchange.using("snippets", function() {
createEditor();
});
}
else {
createEditor();
}
});

function createEditor() {
StackExchange.prepareEditor({
heartbeatType: 'answer',
convertImagesToLinks: true,
noModals: true,
showLowRepImageUploadWarning: true,
reputationToPostImages: 10,
bindNavPrevention: true,
postfix: "",
imageUploader: {
brandingHtml: "Powered by u003ca class="icon-imgur-white" href="https://imgur.com/"u003eu003c/au003e",
contentPolicyHtml: "User contributions licensed under u003ca href="https://creativecommons.org/licenses/by-sa/3.0/"u003ecc by-sa 3.0 with attribution requiredu003c/au003e u003ca href="https://stackoverflow.com/legal/content-policy"u003e(content policy)u003c/au003e",
allowUrls: true
},
noCode: true, onDemand: true,
discardSelector: ".discard-answer"
,immediatelyShowMarkdownHelp:true
});


}
});














draft saved

draft discarded


















StackExchange.ready(
function () {
StackExchange.openid.initPostLogin('.new-post-login', 'https%3a%2f%2fmath.stackexchange.com%2fquestions%2f3003072%2fsuppose-h%25e2%25a9%25bdg-prove-that-if-h-g-e-then-h-g-e%23new-answer', 'question_page');
}
);

Post as a guest















Required, but never shown






























active

oldest

votes













active

oldest

votes









active

oldest

votes






active

oldest

votes
















draft saved

draft discarded




















































Thanks for contributing an answer to Mathematics Stack Exchange!


  • Please be sure to answer the question. Provide details and share your research!

But avoid



  • Asking for help, clarification, or responding to other answers.

  • Making statements based on opinion; back them up with references or personal experience.


Use MathJax to format equations. MathJax reference.


To learn more, see our tips on writing great answers.





Some of your past answers have not been well-received, and you're in danger of being blocked from answering.


Please pay close attention to the following guidance:


  • Please be sure to answer the question. Provide details and share your research!

But avoid



  • Asking for help, clarification, or responding to other answers.

  • Making statements based on opinion; back them up with references or personal experience.


To learn more, see our tips on writing great answers.




draft saved


draft discarded














StackExchange.ready(
function () {
StackExchange.openid.initPostLogin('.new-post-login', 'https%3a%2f%2fmath.stackexchange.com%2fquestions%2f3003072%2fsuppose-h%25e2%25a9%25bdg-prove-that-if-h-g-e-then-h-g-e%23new-answer', 'question_page');
}
);

Post as a guest















Required, but never shown





















































Required, but never shown














Required, but never shown












Required, but never shown







Required, but never shown

































Required, but never shown














Required, but never shown












Required, but never shown







Required, but never shown







Popular posts from this blog

QoS: MAC-Priority for clients behind a repeater

Ивакино (Тотемский район)

Can't locate Autom4te/ChannelDefs.pm in @INC (when it definitely is there)